. (3 points) for a simple linear regression, if the sum of squares for error (sse) is 40 and the sum of squares due to the model (ssm) is 60, what is ? (a) 1.50 (b) 0.40 (c) 0.60 (d)

Answers

Answer 1

If the sum of squares for error (SSE) is 40 and the sum of squares due to the model (SSM) is 60, therefore, the answer is (c) 0.60.

Based on your question,  the coefficient of determination (R²) for a simple linear regression, given the sum of squares for error (SSE) is 40 and the sum of squares due to the model (SSM) is 60.

To calculate R², follow these steps:

1. Calculate the total sum of squares (SST): SST = SSE + SSM
2. Divide SSM by SST: R² = SSM / SST

Now, let's apply the values from your question:
To calculate, we use the formula:
= SSM / SSM + SSE)

Plugging in the given values, we get:
= 60 / (60 + 40) = 0.6
SST = SSE + SSM = 40 + 60 = 100
R² = SSM / SST = 60 / 100 = 0.60

So, the coefficient of determination (R²) is 0.60, which corresponds to option (c).

Learn more about Squares:

brainly.com/question/28776767

#SPJ11


Related Questions

Rounding up the number of contributions, for how long has William been making end-of-quarter contributions of $1200 to his RRSP if the RRSP has earned 4.75% compounded annually and is presently worth $74,385?

Answers

William has been making end-of-quarter contributions of $1200 for 11years 8months

What is quarter?

25 percent of anything is equal to one-quarter of it. This is how many athletic events are split up, and sports commentators frequently use phrases like "Here is the score at the end of the first quarter."

One-fourth is referred to as a quarter. Each of you will consume one-fourth of a pizza if it is divided into four pieces and shared with three buddies.

Given:

Final value (FV)= $74,385

Payment(PMT) = $1200

Present value(PV)= $0

So,

Interest rate(r)= 4.75/4= 1.1875%

By putting the value of each and using the financial calculator we get,

N= 46.729

Or, N≅46.73

So the number of years= 46.73/4= 11.68 years

Months= .68*12= 8.16months

Then the correct answer is 11years 8months

To learn more about quarter from the given link

https://brainly.com/question/382406

#SPJ4

8.06 Finding Side Lengths of Triangles

Answers

a² + b² = c² is true for the first triangle but false for the second triangle.

What is Pythagorean theorem?

In Mathematics and Geometry, Pythagorean's theorem is modeled or represented by the following mathematical equation (formula):

a² + b² = c²

Where:

a, b, and c represents the length of sides or side lengths of any right-angled triangle.

By substituting the given parameters into the formula for Pythagorean's theorem, we have the following;

a² + b² = c²

4² + 2² = c²

c² = 16 + 4

c = √20 or 2√5 units.

a² + b² = c²

5² + 2² = (√45)²

45 = 25 + 9

45 = 34 (False).

Read more on Pythagorean theorem here: brainly.com/question/15430861

#SPJ1

The radius of a circle is 7 miles. What is the area of a sector bounded by a 180° arc?

Answers

The area of the 180° sector is  76.93 mi ²

How to find the area of the sector?

The area of a circle of radius R is given by the formula:

A = pi*R²

Where pi = 3.14

Here the radius is 7 mi, and we want a section of 180°. Remember that the angle in a circle is 360°, then a section of 180° is half a circle, then the area is 0.5 times the one written above.

Then the area will be:

A = 0.5*3.14(7 mi)²

A = 76.93 mi ²

Learn more about circles at:

https://brainly.com/question/1559324

#SPJ1

write the equation of a circle given the center and radius. write the equation of a circle whose center is (4, -5) and which has a radius of 3.

Answers

The equation of a circle can be written as[tex](x - h)^2 + (y - k)^2 = r^2,[/tex]where (h, k) is the center of the circle and r is the radius.

Circle's basic equation is represented by:

[tex](x-h)^2 + (y-k)^2 = r^2[/tex]

where the radius is r and the circle's center's coordinates are (h,k).

Let's first consider what a circle is before determining its equation. A circle is a collection of all points in a plane that is uniformly distanced from a fixed point. The fixed point is referred to as the circle's center. The radius of a circle is the separation between the center and any point along its circumference. In this post, we'll go over what a circle equation in standard form is and how to calculate a circle's equation when the origin serves as its center.
Therefore, the equation of a circle whose center is (4, -5) and which has a radius of 3 is:

[tex](x - 4)^2 + (y + 5)^2 = 3^2[/tex]
Simplifying and expanding the equation gives:

[tex](x - 4)^2 + (y + 5)^2 = 9[/tex]

And that is the equation of the circle.

learn more about equation of a circle

https://brainly.com/question/29288238

#SPJ11

Find the area of each quadrilateral. Round answers to the nearest tenth.

Answers

The area of each of the quadrilateral is calculated as:

11. 48 square meters;  12. 144.3 square centimeters;   13. 8.2 square yards.

14. 132 square yds;  

How to Find the Area of Each Quadrilateral?

The area of each quadrilateral = height * base/width

11. Height = 6 m

Base = 8 m

Area= 6 * 8 = 48 square meters.

12. Height = 13 cm

Base = 11.1 cm

Area= 11.1 * 13 = 144.3 square centimeters.

13. Height = 4.1 yd

Base = 2 yd

Area= 2 * 4.1 = 8.2 square yards.

14. Height = 12 yd

Base = 11 yd

Area= 11 * 12 = 132 square yds

Learn more about the area of quadrilateral on:

https://brainly.com/question/31543314

#SPJ1

For numbers 7-9, write the function rule for the given situation.


7. A landfill has 50,000 tons of waste in it. Each month it accumulates an average of 420 more
tons of waste. What is a function rule that represents the total amount of waste after m months?
f(m) =


8. A kennel charges $15 per day to board dogs. Upon arrival, each dog must have a flea bath that
costs $12. Write a function rule for the total cost for n days of boarding plus a bath.
f(n)=


9. A worker's earnings are a function of the number of hours ʼn worked at a rate of $10.75 per
hour. Write a function rule for the total amount of money the worker makes for h hours.
f(h) =

Answers

7. f(m) = 50,000 + 420m

8. f(n) = 15n + 12

9. f(h) = 10.75h

For positive constants k and g, the velocity, v, of a particle of mass m at time t is given by v= (mg/k)(1-e^(-kt/m)) At what rate is the velocity is changing at time 0? At t=7? What do your answers tell you about the motion? At what rate is the velocity changing at time 0? rate= At what rate is it changing at t=7? rate =

Answers

This indicates that the particle is accelerating due to gravity. At t=7, the rate at which the velocity is changing depends on the value of k, m, and g. This implies that the particle's acceleration may vary depending on these constants and time.

At time 0, the rate at which the velocity is changing can be found by taking the derivative of the velocity function with respect to time, t.

v(t) = (mg/k)(1-e^(-kt/m))

v'(t) = (mg/k)((ke^(-kt/m))/m)

Plugging in t=0 gives:

v'(0) = (mg/k)((k)/m) = g

Therefore, the rate at which the velocity is changing at time 0 is g.

At t=7, the rate at which the velocity is changing can also be found by taking the derivative of the velocity function with respect to time, t, and plugging in t=7:

v'(7) = (mg/k)((ke^(-7k/m))/m)

This value will depend on the specific values of k, g, and m that are not given in the question.

These rates of change tell us about the motion of the particle. If the rate of change of velocity is positive, the particle is accelerating. If it is negative, the particle is decelerating. If it is zero, the particle is moving at a constant velocity.

Learn more about value here:

brainly.com/question/13961297

#SPJ11

Lauren predicts that 92% of the people she invites to her party will come. If she wants to have at least 23 guests, how many people should she invite to her party?

A. 21 people
B. 134 people
C. 250 people
D. 25 people

Answers

Lauren should invite at least 25 people to her party in order to have at least 23 guests.

Option D is the correct answer.

We have,

Let x be the number of people Lauren should invite to her party.

Since 92% of the people she invites are expected to come, the actual number of guests she can expect is 0.92x.

We want to find the value of x that makes the expected number of guests at least 23.

Therefore, we can set up the following inequality:

0.92x ≥ 23

Solving for x, we divide both sides by 0.92:

x ≥ 25

Therefore,

Lauren should invite at least 25 people to her party in order to have at least 23 guests.

Learn more about inequalities here:

https://brainly.com/question/20383699

#SPJ1

The Occupancy Problem. You have n bins and m balls. For each ball, you will throw it into one of the bins uniformly at random. Thus, if we define Aij to be the event that ball i lands in bin j, then: Pr[Aij] = 1/n Let Y; be a random variable representing the number of balls you need to throw to hit the next empty bin after j bins were filled (with at least one ball per bin). Now the goal is to find out the expected number of balls Y = j=1Σm-Yj, you need to throw until every bin becomes non-empty (i.e. every bin contains at least one ball). • Assuming that there are i non-empty bins so far, what is the probability that the next ball you throw will fall into an empty bin? • Find E[Y] • Show that E[Y] = O(n lg n) • Using Markov's Ine quality, find an upper bound on Pr[Y ≥ O(n³ lg n³)]

Answers

we can choose k = (m + Hn-1)/(O(n^3 log n^3)) to obtain the desired upper bound on Pr[Y ≥ k].

Assuming there are i non-empty bins so far, the probability that the next ball you throw will fall into an empty bin is (n-i)/n. This is because there are n bins in total, and i of them are already non-empty. Therefore, there are (n-i) empty bins left, and each has an equal probability of receiving the ball.

To find E[Y], we can use linearity of expectation. Let Yi be the number of balls we need to throw to hit the next empty bin after i bins have been filled. Then, we have:

Y = Y1 + Y2 + ... + Yn

We know that Y1 = m, since we need to throw m balls to fill the first bin. For i > 1, we have:

E[Yi] = 1/(n-i+1) + E[Yi-1]

The first term represents the probability of hitting an empty bin on the next throw, and the second term represents the expected number of throws we need after that happens. We can solve this recurrence relation by substitution:

E[Y2] = 1/(n-1) + E[Y1] = 1/(n-1) + m

E[Y3] = 1/(n-2) + E[Y2] = 1/(n-2) + 1/(n-1) + m

E[Y4] = 1/(n-3) + E[Y3] = 1/(n-3) + 1/(n-2) + 1/(n-1) + m

...

We can see a pattern emerging, where each term has an additional 1/(n-i) compared to the previous term. Therefore, we have:

E[Y] = m + 1/(n-1) + 1/(n-2) + ... + 1/n

= m + Hn-1

≈ m + ln(n) (where Hn-1 is the (n-1)th harmonic number)

To show that E[Y] = O(n log n), we can use the fact that the harmonic series diverges, but the sum of the first n terms is bounded by ln(n) + 1. Therefore, we have:

E[Y] ≈ m + ln(n) ≤ m + ln(n) + 1

= m + O(log n)

Since m is a constant, we can say that E[Y] = O(n log n).

Using Markov's inequality, we have:

Pr[Y ≥ k] ≤ E[Y]/k

Therefore, we want to find k such that:

E[Y]/k ≤ O(n^3 log n^3)

Substituting in the expression we found for E[Y], we have:

(m + Hn-1)/k ≤ O(n^3 log n^3)

Rearranging, we get:

k ≥ (m + Hn-1)/(O(n^3 log n^3))

Therefore, we can choose k = (m + Hn-1)/(O(n^3 log n^3)) to obtain the desired upper bound on Pr[Y ≥ k].

To learn more about Continuing visit:

https://brainly.com/question/31694381

#SPJ11

Graph and create a table
(Show your work step by step please)

Answers

The value of given function [tex]f(x) = \frac{4}{x+2}+2[/tex] at x = -4 is 0, and at x = 4 is 2.17. The graph and table is attached below.

To graph and create a table for f(x) = 4/(x+2) + 2, we can start by making a table of values. To calculate the value of F(x) at each given x, we substitute the value of x into the function and simplify as follows,

At x = -4

F(x) = (4/(-4+2)) + 2 = -2 + 2 = 0

Therefore, F(-4) = 0.

At x = -3

F(x) = (4/(-3+2)) + 2 = undefined (division by zero)

Therefore, F(-3) is undefined.

At x = -2

F(x) = (4/(-2+2)) + 2 = undefined (division by zero)

Therefore, F(-2) is undefined.

At x = -1

F(x) = (4/(-1+2)) + 2 = 6

Therefore, F(-1) = 6.

At x = 0

F(x) = (4/(0+2)) + 2 = 10

Therefore, F(0) = 10.

At x = 1

F(x) = (4/(1+2)) + 2 = 4

Therefore, F(1) = 4.

At x = 2

F(x) = (4/(2+2)) + 2 = 2.67

Therefore, F(2) = 2.67.

At x = 3

F(x) = (4/(3+2)) + 2 = 2.4

Therefore, F(3) = 2.4.

At x = 4

F(x) = (4/(4+2)) + 2 = 2.17

Therefore, F(4) = 2.17.

Now we can plot these points on a coordinate plane and connect them to create the graph.

A vertical asymptote is a vertical line on a graph that the function approaches but never touches or crosses. It occurs when the denominator of a rational function (a function with a fraction of polynomials) becomes zero and the function becomes undefined at that point.

A horizontal asymptote is a horizontal line on a graph that the function approaches as x approaches positive or negative infinity. It describes the long-term behavior of a function as x becomes very large or very small.

We can see from the graph that there is a vertical asymptote at x = -2, and a horizontal asymptote at y = 2.

To know more about graph:

https://brainly.com/question/13819784

#SPJ1

What’s the answer I need help asap?

Answers

a) The function are described as follows:

y= sin( x) - odd

y= cos (x) - even

y = tan (  x) - neither even or odd

b) y= sin (x) - symmetric about the origin

 y  = cos (x) - symmetric about tthe y-axis

y = tan (x) - does not have a point or line of symmetry


Why is this so ?

A) The following definitions can be used to  determine if a function is even or odd ...

If f(-x  ) = f (x ) for every x in the domain, the function is even.

If f(-x) =  -  f(x) for every x in the domain, t e function is odd

Using these definitions  , we can examine the following functions

y =  sin (x)

Because sin   (-x) = -sin(x) for any angle x, the function is odd.

y = cos  (x)

Cos  (-x) = cos (x) for any angle x, hence the function is even.

y = tan( x)

Tan(-x) = -tan(x) for every angle x.   Only if x does not equal ( n + 0.5), where n   is an integer, is the function even or odd.

B)

The line or  point of symmetry of a function is detrmined by whether it is even or odd...

The y -axis (x=0) is the line of symmetry for an even function.

The origin (0,0) is the point of symmetry for an odd function.

y = sin(x)     his is an odd function, so the point of symmetry is the origin (0,0).

y = cos(x) his is an even function, so the line of symmetry is the y-axis (x=0).

y = tan(x)

This is neither even nor odd, so it does not have a line or point of symmetry.

Learn more about lines of symmetry:
https://brainly.com/question/30963765
#SPJ1

understand subtraction as an unknown-addend problem. for example, subtract 10-8 by finding the number that makes 10 when added to 8.

Answers

To understand subtraction as an unknown-addend problem means to view subtraction as finding the missing number in an addition problem.

In the example given, subtracting 10-8 means finding the number that needs to be added to 8 to make 10. This is essentially an addition problem with an unknown addend.

To solve the problem, one needs to think about what number added to 8 will result in 10. This can be done by counting up from 8 until you reach 10, which is a difference of 2. Therefore, 10-8=2, and the missing number is 2.

In general, to subtract any two numbers using this method, you can start with the smaller number and count up until you reach the larger number. The difference between the two numbers is the missing addend. For example, to subtract 15-7, you would start with 7 and count up to 15, which is a difference of 8. So 15-7=8.

To learn more about Number - brainly.com/question/17429689?

#SPJ11

In these activities, we use the following applet to select a random sample of 8 students from the small college in the previous example. At the college, 60% of the students are eligible for financial aid. For each sample, the applet calculates the proportion in the sample who are eligible for financial aid. Repeat the sampling process many times to observe how the sample proportions vary, then answer the questions. Use the applet to select a random sample of 8 students. Repeat to generate many samples. The applet gives the sample proportion for each sample. Examine the variability in the sample proportions you generated with the applet. Which of the following sequences of sample proportions is most likely to occur for 5 random samples of 8 students from this population?

Answers

The sequence of sample proportions (0.625, 0.563, 0.750, 0.500, 0.625) falls within this range and is the most likely to occur for 5 random samples of 8 students from this population.

In this question, we are given a small college population where 60% of students are eligible for financial aid. We use an applet to select a random sample of 8 students from the population, and the applet calculates the proportion in the sample who are eligible for financial aid. We repeat this process many times to observe how the sample proportions vary.

To answer this question, we need to understand the concept of sampling variability. In statistics, sampling variability refers to the fact that different random samples from the same population can yield different results. The variability in sample results is due to chance and can be quantified using statistical measures such as the standard deviation.

The question asks us to examine the variability in the sample proportions generated by the applet and select the most likely sequence of sample proportions for 5 random samples of 8 students from the population.

Based on the concept of sampling variability, we can expect the sample proportions to vary from sample to sample. However, we can make some predictions about the range of values that the sample proportions are likely to fall within. Specifically, we can use the formula for the standard error of the proportion:

SE(p) = sqrt[p(1-p)/n]

where p is the population proportion, n is the sample size, and sqrt denotes the square root function.

Using this formula, we can calculate that the standard error of the proportion for a sample of 8 students from a population where 60% are eligible for financial aid is:

SE(p) = sqrt[0.6(1-0.6)/8] = 0.165

This means that we can expect the sample proportions to vary by approximately plus or minus 0.165 around the true population proportion of 0.6. Therefore, any sequence of sample proportions that falls within this range is a plausible outcome.

Looking at the options provided, the sequence of sample proportions (0.625, 0.563, 0.750, 0.500, 0.625) falls within this range and is the most likely to occur for 5 random samples of 8 students from this population.

To learn more about Specifically visit:

https://brainly.com/question/11297584

#SPJ11

find an equation of the line of intersection of the following 2 planes: and use vector form for the equation and use to represent the parameter.

Answers

To find the equation of the line of intersection of two planes, we need to find the direction vector of the line. This can be done by taking the cross product of the normal vectors of the two planes.

Let the two planes be:
P1: 2x - y + 3z = 5
P2: x + 2y - 4z = -1
The normal vectors of these planes are:
n1 = <2, -1, 3>
n2 = <1, 2, -4>
Taking the cross product of these two vectors, we get:
n1 x n2 = <14, 10, 5>
This is the direction vector of the line of intersection.
To get the vector form of the equation, we need a point on the line. We can choose any point that lies on both planes. To make it easy, we can set z = 0 in both planes and solve for x and y.
From P1:
2x - y = 5
From P2:
x + 2y = -1
Solving these equations, we get:
x = -7/5
y = -3/5
So a point on the line is (-7/5, -3/5, 0).
Using this point and the direction vector, the vector form of the equation of the line of intersection is:
r = <-7/5, -3/5, 0> + t<14, 10, 5>
Here, t represents the parameter.

Learn more about direction vector here:

https://brainly.com/question/12002625

#SPJ11

Seth bought a pair of shorts. The original cost was $21, but the store was having a sale of 25% off. Seth also had a coupon for 15% off any purchase at checkout. How much did Seth pay for the pair of shorts?

Answers

Answer: $13.39

Step-by-step explanation:

first, you take 25% off 21, by multiplying 21*.25 which is 5.25

next, subtract 5.25 from 21, which gets you 15.75

next, add the 15% off coupon, by multiplying 15.75*.15 which is 2.3625

last, subtract 2.3625 from 15.75, which gets you 13.3875, or $13.39 rounded

Inbrahim draws the image below onto a card. He then copies the same image onto the same different cards. If he draws 70 triangles in total, how many circles does he draw

Answers

The number of circles he will draw is 30 circles.

What is Algebra?

The branch of mathematics which involves the study and manipulation of mathematical symbols is known as Algebra. This field comprises the use of characters and signs to symbolize unknown values and their linkages.

How to solve:

On one card, we have 14 triangles and 6 circles

Therefore, if we have 70 triangles,

number of cards= 70/14

= 5 card

Thus, Total number of circles is 6 x 5 = 30 circles

Read more about algebra here:

https://brainly.com/question/22399890

#SPJ1

please answer
Transform into a rectangular form of complex numbers: 5<30°

Answers

The rectangular form of the complex number 5<30° can be found using the following formula:

a + bi = r(cosθ + i sinθ)

where a and b are the real and imaginary parts of the complex number, r is the modulus or magnitude of the complex number, and θ is the argument or angle of the complex number.

In this case, we have:

r = 5 (the modulus or magnitude)
θ = 30° (the argument or angle)

Using the formula, we can find the rectangular form as follows:

a + bi = 5(cos30° + i sin30°)

a + bi = 5(√3/2 + i/2)

a + bi = (5/2)√3 + (5/2)i

Therefore, the rectangular form of the complex number 5<30° is (5/2)√3 + (5/2)i.


#SPJ11

Learn more on: https://brainly.com/question/21664240

Kevin drew a double number line diagram and stated of 24 is 15. Is he correct?

Answers

Yes Kevin's comparison in the number line is correct because 10.9 is less than 11.5.

What is Number line?

A number line is described as a picture of a graduated straight line that serves as visual representation of the real numbers.

We can agree with Kevin  after referencing the decimal value chart or decimal comparisons below.

o   t     h

10  9

11   5

A number line can also be referred to as a  pictorial representation of numbers on a straight line that is used for  comparing and ordering numbers.

The complete question is attached in the image.

Learn more about number line at:

https://brainly.com/question/24644930

#SPJ1

A=

1 1

0 1

Calculate A2, A3, A4,. . . Until you detect a pattern. Write a general formula for An

Answers

Answer:

  [tex]A_n=\left[\begin{array}{cc}1&n\\0&1\end{array}\right][/tex]

Step-by-step explanation:

You want the general formula for the n-th power of matrix A, where ...

  [tex]A=\left[\begin{array}{cc}1&1\\0&1\end{array}\right][/tex]

Sequence

The sequence of powers A, A², A³, A⁴ is shown in the attachment. It strongly suggests that the upper right element of the matrix is equal to the power.

The formula for An is ...

  [tex]\boxed{A_n=\left[\begin{array}{cc}1&n\\0&1\end{array}\right]}[/tex]

The petrol consumption of a van, in litres per 100 kilometres, is given by the formula
Petrol consumption=100×litres of petrol used/kilometres travelled
Imran used his van to travel 250 kilometres,correct to 3 significant figures
The van used 21.3 litres of petrol, correct to 3 significant figures
Imran says, "My van used less than 8.5 litres of petrol per 100 kilometres."
Could Imran be wrong Yes/No

Answers

The requried, value is less than 8.5, as Imran claimed. Therefore, he is correct and not wrong.

To verify this, we can use the formula given:

Petrol consumption = 100 × litres of petrol used / kilometres travelled

Substituting the given values, we get:

Petrol consumption = 100 × 21.3 / 250 = 8.52

Rounding this value to three significant figures, we get:

Petrol consumption = 8.52

This value is indeed less than 8.5, as Imran claimed. Therefore, he is correct and not wrong.

Learn more about the rate here:

https://brainly.com/question/4782182

#SPJ1

Grace bought a pair of pants on sale for $24, which is 60% off the original price. What was the original price of the pants?

Answers

The original price of the pants be $40.

We have,

Grace bought a pair of pants on sale for $24.

This is 60% of original price.

let the original price be x.

So, 60% of x = 24

60/100 x =24

x = 2400/60

x = $40

Thus, the original price be $40.

Learn more about Algebra here:

https://brainly.com/question/24875240

#SPJ1

Use properties of logarithms

PLEASE HELP SOLVE 30 PTS

Answers

The expanded logarithmic expression of log_b (x²y)/z² is 2 log_b x + log_b y - 2 log_b z.

Expanding the given logarithmic expression using the properties of logarithms, we get:

log_b (x²y)/z² = log_b x²y - log_b z²

Using the power rule of logarithms, we can simplify log_b x²y as:

log_b x²y = log_b x² + log_b y

Then, using the quotient rule of logarithms, we can simplify log_b z² as:

log_b z² = 2log_b z

Substituting these simplifications in the original expression, we get:

log_b (x²y)/z² = log_b x² + log_b y - 2log_b z

This is the expanded form of the given logarithmic expression using the properties of logarithms.

Learn more about the properties of logarithms at

https://brainly.com/question/30226560

#SPJ4

The question is -

Use properties of logarithms to expand the following logarithmic expression as much as possible.

log_b (x²y)/z²

How many points of inflection will f(x) = 3x^7 + 2x^5 - 5x - 12 have

a 4

b 5

c 2

d 3

Answers

There is only one point of inflection. Answer: d) 3

The second derivative of the function f(x) is:

[tex]f''(x) = 126x^5 + 40x^3 - 5[/tex]

The second derivative of a function is the derivative of its first derivative. It is denoted represents the rate of change of the slope of the function.

In other words, if the first derivative f'(x) represents the slope of the function, the second derivative f''(x) represents the rate at which the slope is changing.

The points of inflection occur where the concavity changes, that is where the second derivative changes sign or equals zero.

Setting f''(x) = 0, we have:

[tex]126x^5 + 40x^3 - 5 = 0[/tex]

This equation has only one real solution, which can be found numerically:

x ≈ 0.357

Therefore, there is only one point of inflection. Answer: d) 3

To know more about differential here

https://brainly.com/question/1164377

#SPJ4

by how much would the range decrease if the number 4 replaced the number 5 in the set

Answers

The range will decrease by 3.

How to explain the range

The range of a set of data is the difference between the largest and smallest values.

Set of numbers: 9, 5, 1, 7, 4, 4, 7, 9

Order: 1, 4, 4, 5, 7, 7, 9, 9

Range: 9 - 1 = 8

If the number 7 replaced the number 1 in the set

Order: 4, 4, 5, 7, 7, 7, 9, 9

Range: 9 - 4 = 5

How much would the range decrease if the number 7 replaced the number 1 in the set

8 - 5 = 3

So, the range decrease by 3 if the number 7 replaces the number 1 in the set.

Learn more about range on

https://brainly.com/question/26098895

#SPJ1

What is equal in the value to 27%?
F. 2. 7
G. 0. 027
H. 0. 270
J. 0. 27

Answers

H. 0.270 is equal in value to 27% in the given case. Option 3 is correct

A percentage is a way of expressing a number as a fraction of 100. The symbol for percentage is the percent sign (%).

To convert a percentage to a decimal, we divide the percentage by 100. For example, 50% is equal to 0.50 as a decimal.

To convert a percentage to a decimal, we divide the percentage by 100.

So to convert 27% to a decimal, we divide 27 by 100:

27/100 = 0.27

Therefore, 27% is equal to 0.27 as a decimal.

To know more about percentage here

https://brainly.com/question/24877689

#SPJ4

Your school wants to take out an ad in the paper congratulating the basketball team on a successful​ season, as shown to the right. The area of the photo will be half the area of the entire ad. What is the value of​ x?

Answers

The value of x that makes the photo area half of the entire area is: 1.12 in

How to solve Algebra Word Problems?

The area of a rectangle is given by the formula:

A = L * w

where:

L is length

w is width

Thus:

Area of photo = 4 * 2 = 8 in²

We are told that this area is half of the entire ad. Thus:

¹/₂(4 + x)(2 + x) = 8

x² + 6x + 8 = 16

x² + 6x - 8 = 0

Solving using a quadratic calculator gives:

x = 1.12 in

Read more about Algebra Word Problems at: https://brainly.com/question/21405634

#SPJ1

A cooler is filled with 4 1/2 gallons of water

Answers

A Total of 144 small cups can be filled with the water from the cooler before it's empty.

The cooler has 4 1/2 gallons of water, which can be written as 9/2 gallons. Each small cup holds 1/32 gallon. To find the number of small cups that can be filled with the water from the cooler, we need to divide the total amount of water by the amount of water in each cup.

9/2 ÷ 1/32 = (9/2) * (32/1) = 144 small cups.

Therefore, 144 small cups can be filled with the water from the cooler before it's empty.

Learn more about Simple Maths

https://brainly.com/question/30198551

#SPJ4

Complete Question:

A cooler is filled with 4 1/2 gallons of water. There are small cups that each hold 1/32 gallon.

How many small cups can be filled with the water from the cooler before it's empty?

Two dice are rolled. Find the probability of the following event. The first die is 6 or the sum is 8. The probability of the event "the first die is 6 or the sum is 8" is (Type an integer or a simplified fraction.)

Answers

To find the probability of the event "the first die is 6 or the sum is 8," we need to count the number of outcomes that satisfy this event and divide it by the total number of possible outcomes.


To find the probability of the event "the first die is 6 or the sum is 8," we need to consider the total possible outcomes when rolling two dice and the favorable outcomes for this event.

Total possible outcomes: 6 sides on each die, so there are 6 x 6 = 36 possible outcomes.

Favorable outcomes:
1. First die is 6: There are 6 possible outcomes (6,1), (6,2), (6,3), (6,4), (6,5), and (6,6).
2. Sum is 8: There are 5 possible outcomes (2,6), (3,5), (4,4), (5,3), and (6,2).

However, (6,2) is counted twice, so we should subtract 1 from the total favorable outcomes: 6 + 5 - 1 = 10.

Probability = Favorable outcomes / Total possible outcomes = 10/36. Simplifying the fraction gives 5/18.

So, the probability of the event "the first die is 6 or the sum is 8" is 5/18.

To learn more about probability, click here:

brainly.com/question/30034780

#SPJ11

Suppose speeds of vehicles on a particular stretch of roadway are normally distributed with mean 36.6 mph and standard deviation 1.7 mph. A. Find the probability that the speed X of a randomly selected vehicle is between 35 and 40 mph. B. Find the probability that the mean speed of 20 randomly selected vehicles is between 35 and 40 mph.

Answers

The probability that the mean speed of 20 randomly selected vehicles is between 35 and 40 mph is approximately 0.99997

A. To find the probability that the speed X of a randomly selected vehicle is between 35 and 40 mph, we need to standardize the values and use the standard normal distribution table.

We can standardize the values as follows:

[tex]z1 = \frac{(35 - 36.6)}{1.7} = -0.94[/tex]

[tex]z2 = \frac{(40 - 36.6)}{1.7} = 2.00[/tex]

Using the standard normal distribution table, we find the probability that a standard normal variable is between -0.94 and 2.00 to be approximately 0.7794.

B. To find the probability that the mean speed of 20 randomly selected vehicles is between 35 and 40 mph, we need to use the central limit theorem.

The central limit theorem tells us that the distribution of sample means will be approximately normal, with mean equal to the population mean and standard deviation equal to the population standard deviation divided by the square root of the sample size.

Thus, the mean of the sampling distribution of the sample means is:

u-X=u=36.6

And the standard deviation of the sampling distribution of the sample means is:

[tex]\frac{ σ}{\sqrt{n} } = \frac{1.7}{\sqrt{20} } = 0.3808[/tex]

We can standardize the values using the formula:

[tex]z1 =\frac{35-36.6}{0.3808} = -4.21[/tex]

[tex]z2 =\frac{40-36.6}{0.3808} = 8.92[/tex]

we can find the probability that the standard normal variable is between -4.21 and 8.92 by finding the probability that it is greater than -8.92 (which is essentially 0) and subtracting the probability that it is greater than 4.21 from 1:

P(-4.21 < Z < 8.92) = 1 - P(Z > 4.21) = 1 - 0.00003 = 0.99997

Therefore, the probability that the mean speed of 20 randomly selected vehicles is between 35 and 40 mph is approximately 0.99997.

To know more about "Probability" refer here:

https://brainly.com/question/30034780#

#SPJ11

What are the discontinuity and zero of the function f(x) = x^2+5x+6/x+2

Answers

The discontinuity of the given function is at (−2, 1) and zero at (−3, 0).

The given function is:

f(x) =   [tex]\frac{x^{2} + 5x + 6}{x + 2}[/tex]

We will factorize the numerator and then reduce this function.

= [tex]\frac{x^{2} + 2x + 3x + 6}{x + 2}[/tex]

=  [tex]\frac{x(x + 2) +3 (x + 2)}{x + 2}[/tex]

= [tex]\frac{(x + 2) (x + 3)}{x + 2}[/tex]

If we take the value of x as -2, both the numerator and denominator will be 0. Note that for x = -2, both the numerator and denominator will be zero. When both the numerator and denominator of a rational function become zero for a given value of x we get a discontinuity at that point. which means there is a hole at x = -2.

Now, when we reduce this function by canceling the common factor from the numerator and denominator we get the expression f(x) = x + 3.  If we use the value of x = -2 in the previous expression we get;

f(x) = x + 3 =  = -2 + 3

f(x) = 1

Therefore, there is a discontinuity (hole) at (-2, 1).

If x = -3, the value of the function is equal to zero. This means x = -3 is a zero or root of the function.

Therefore, (-3, 0) is a zero of the function.

To learn more about discontinuity;

https://brainly.com/question/9837678

#SPJ4

Other Questions
pasadena candle inc. pays 40% of its purchases on account in the month of the purchase and 60% in the month following the purchase. if purchases are budgeted to be $40,000 for august and $36,000 for september.prepare a simple cash budget for pasadena candle inc. what is the most likely reason for the poor appearance of the bacteria on this slide when sellers advertise a low price for an item generally unavailable to the consumer and then push the consumer to buy a more expensive item, they are engaging in ______ advertising. In a divorce, to obtain custody, the genetic father has to convince the court of his fitness and ability to care for his child, whereas if the mother decides to keep her child, she need not legally demonstrate her fitness (True or False) 1. An alphabetic listing of the names and GPAs of everyone in your class is an example of data. Where do erasers go for vacation? missing lengths Draw the Voronoi tile for the following data set (-1,-1), (-1,1), (1,-1), (1,1),(0,0) around the Assume that the point(0,0) has a classification and all the other points have a + classification. If we use 1-nearest neighbors, what will be probability of a point chosen uniformly at random in the region -1 5xs1,-15 y s 1 to be classified as '+'? [5 Marks) Select the table of values where the quadratic function changes direction at a different value of x than the others. A. x -3 -2 -1 0 1 2 3 y 4 3 4 7 12 19 28 B. x -3 -2 -1 0 1 2 3 y 1 -1 1 7 17 31 49 C. x -3 -2 -1 0 1 2 3 y 2 3 2 -1 -6 -13 -22 D. x -3 -2 -1 0 1 2 3 y 28 19 12 7 4 3 4 Reset the ratio of students who prefer pineapple to students who prefer kiwi is 12 to 5. which pair of equivalent ratios could be used to find how many students prefer kiwi if there are 357 total students Under this part of Sprint Planning, the Development Team is more active in planning, and Product Owner is mostly answered and clarify details Please help its due soon What kind of scientist would use a dichotomous key? The ethical issue of voluntary participation is most closely related to which one of the following?a. anonymityb. informed consentc. deceptiond. confidentiality the office of management and budget (omb) and the national security council (nsc)are both parts of the ANAGERIAL ACCOUNTING: AUS Corporation Has Provided The Following Contribution Format Income Statement. All Questions Concern Situations That Are Within The Relevant Range. A. What Is The Degree Of OperatingMANAGERIAL ACCOUNTING:AUS Corporation has provided the following contribution format income statement. All questions concern situations that are within the relevant range.aWhat is the degree of operating leverage?b. Using the degree of operating leverage, what is the estimated percent increase in net operating income of a 15% increase in sales? Question 7 What is the volume of the pyramid? (Round to the nearest tenth)11.2 m11 m8m What was the main reason behind the Japanese attack on Pearl Harbor? What is the best way to display data if there is a narrow range, and we want to see the shape of the data?Multiple ChoiceA. frequency table with intervalsB. frequency tableC. stem-and-leaf plotD. line plot Part a of the drawing shows a bucket of water suspended from the pulley of a well; the tension in the rope is 90. 5 n. Part b shows the same bucket of water being pulled up from the well at a constant velocity. What is the tension in the rope in part b? Suppose marginal propensity to consume (mpc) is 0. 7 and there is a $100 increase in autonomous consumption. Given this information, real gdp will increase by